If K delivers the third speech, any of the following could be the student who makes the fourth speech EXCEPT

medasmx@protonmail.com on December 10, 2021

H seems to be ok for me

R J K H T S doesnt appear to violate any rules

Reply
Create a free account to read and take part in forum discussions.

Already have an account? log in

Jay-Etter on January 22, 2022

Hi,
In your order given, we violate rule one because H J K form a consecutive block of three.
Our inferences for this to see why H can't go fourth would be:
_ _ K H _ _
J can't go second, or fourth, because it can't form a 3 block with K and H. So J would have to go 1 or 6 but it also can't go there by rule 4. So J has nowhere to go and H cannot go fourth.